Get Answers to all your Questions

header-bg qa
Filter By

All Questions

Find the value of \frac{4}{\left ( 216 \right )^{\frac{-2}{3}}}+\frac{1}{\left ( 256 \right )^{\frac{-3}{4}}}+\frac{2}{\left ( 243 \right )^{\frac{-1}{5}}}.

Answer. 214
Solution. We have, \frac{4}{\left ( 216 \right )^{\frac{-2}{3}}}+\frac{1}{\left ( 256 \right )^{\frac{-3}{4}}}+\frac{2}{\left ( 243 \right )^{\frac{-1}{5}}}
We know that
216 = 6.6.6 = 63
256 = 4.4.4.4 = 44
243 = 3.3.3.3.3 = 35
So, \frac{4}{\left ( 216 \right )^{\frac{-2}{3}}}+\frac{1}{\left ( 256 \right )^{\frac{-3}{4}}}+\frac{2}{\left ( 243 \right )^{\frac{-1}{5}}}

= \frac{4}{\left ( 6^{3} \right )^{\frac{-2}{3}}}+\frac{1}{\left ( 4^{4} \right )^{\frac{-3}{4}}}+\frac{2}{\left ( 3 ^{5}\right )^{-\frac{1}{5}}}
= \frac{4}{\left ( 6 \right )^{3\times \frac{-2}{3}}}+\frac{1}{\left ( 4 \right )^{4\times \frac{-3}{4}}}+\frac{2}{\left ( 3 \right )^{5\times \frac{-1}{5}}}
\because \left ( \left ( a \right )^{m} \right )^{n}= \left ( a \right )^{mn}
= \frac{4}{6^{-2}}+\frac{1}{4^{-3}}+\frac{2}{3^{-1}}
= 4 × 62 + 43 + 2 × 3    
\because \frac{1}{\left ( a \right )^{-n}}= \left ( a \right )^{n}                                  
= 4 × 36 + 64 + 6
= 144 + 70
= 214

Hence the answer is 214

 

View Full Answer(1)
Posted by

infoexpert27

Simplify : \left ( 256 \right )^{-\left ( 4^{\frac{-3}{2}} \right )}

Answer. 248
Solution.  Given, \left ( 256 \right )^{-\left ( 4^{\frac{-3}{2}} \right )}
We know that,
256 = 2.2.2.2.2.2.2.2 = 28

\left ( 256 \right )^{-\left ( 4^{\frac{-3}{2}} \right )}  = \left ( 2^{8} \right )^{\left ( -4 \right )\times \left ( -\frac{3}{2} \right )}                                   

\because \left ( \left ( a \right ) ^{m}\right )^{n}= \left ( a \right )^{mn}

=\left ( 2 \right )^{8\times \left ( -4 \right )\times \left ( -\frac{3}{2} \right )}
= 2^{8\times 4\times \frac{3}{2}}= 2^{8\times 2\times 3}= 2^{48}
Hence the answer is 248

View Full Answer(1)
Posted by

infoexpert27

Crack CUET with india's "Best Teachers"

  • HD Video Lectures
  • Unlimited Mock Tests
  • Faculty Support
cuet_ads

If  x= \frac{\sqrt{3}+\sqrt{2}}{\sqrt{3}-\sqrt{2}} and y= \frac{\sqrt{3}-\sqrt{2}}{\sqrt{3}+\sqrt{2}} then find the value of x2 + y2.

Answer.  98
Solution.We use the identity \left ( a+b \right )^{2}= a^{2}+b^{2}+2ab
So, \left ( \sqrt{a}+\sqrt{b} \right )^{2}= a+2\sqrt{ab}+b
x^{2}+y^{2}= \left ( \frac{\sqrt{3}+\sqrt{2}}{\sqrt{3}-\sqrt{2}} \right )^{2}+\left ( \frac{\sqrt{3}-\sqrt{2}}{\sqrt{3}+\sqrt{2}} \right )^{2}
= \frac{3+2\sqrt{6}+2}{3-2\sqrt{6}+2}+\frac{3-2\sqrt{6}+2}{3+2\sqrt{6}+2}
= \frac{5+2\sqrt{6}}{5-2\sqrt{6}}+\frac{5-2\sqrt{6}}{5+2\sqrt{6}}
= \frac{\left ( 5+2\sqrt{6} \right )^{2}+\left ( 5-2\sqrt{6} \right )^{2}}{\left ( 5-2\sqrt{6} \right ){\left ( 5+2\sqrt{6} \right )}}
 = \frac{\left ( 25+20\sqrt{6}+24 \right )+\left ( 25-20\sqrt{6} +24\right )}{25-24}

Using   (a – b) (a + b) = a2– b2= 98
Hence the answer is 98.

 

View Full Answer(1)
Posted by

infoexpert27

If  a= \frac{3+\sqrt{5}}{2} then find the value of a^{2}= \frac{1}{a^{2}}

Answer. 7
Solution.
Given that :- a= \frac{3+\sqrt{5}}{2}

\therefore \frac{1}{a}= \frac{2}{3+\sqrt{5}}
On rationalizing the denominator, we get

\frac{1}{a}= \frac{2\left ( 3-\sqrt{5} \right )}{\left ( 3+\sqrt{5} \right )\left ( 3-\sqrt{5} \right )}
Using   (a – b) (a + b) = a2 – b2

= \frac{6-2\sqrt{5}}{3^{2}-\sqrt{5}^{2}}                

= \frac{6-2\sqrt{5}}{9-5}

= \frac{6-2\sqrt{5}}{4}
= \frac{3-\sqrt{5}}{2}
Also, \left ( a+\frac{1}{a} \right )^{2}= a^{2}+\frac{1}{a^{2}}+2
Substituting the values of a and \frac{1}{a}
We get, \left ( \frac{3+\sqrt{5}}{2}+\frac{3-\sqrt{5}}{2} \right )^{2}= \left ( a^{2}+\frac{1}{a^{2}}+2 \right )
\therefore a^{2}+\frac{1}{a^{2}}+2= \left ( \frac{3+\sqrt{5}+3-\sqrt{5}}{2} \right )^{2}
= (3)2 = 9

\therefore a^{2}+\frac{1}{a^{2}}= 9-2= 7
Hence the correct answer is 7.

View Full Answer(1)
Posted by

infoexpert27

JEE Main high-scoring chapters and topics

Study 40% syllabus and score up to 100% marks in JEE

If \sqrt{2}= 1\cdot 414 and \sqrt{3}= 1\cdot 732 then find the value of \frac{4}{3\sqrt{3}-2\sqrt{2}}+\frac{3}{3\sqrt{3}+2\sqrt{2}}

Answer.     2.0632
Solution.   Given that :
\sqrt{2}= 1\cdot 414,\sqrt{3}= 1\cdot 732
\frac{4}{3\sqrt{3}-2\sqrt{2}}+\frac{3}{3\sqrt{3}+2\sqrt{2}}
= \frac{4\left ( 3\sqrt{3}+2\sqrt{2} \right )}{\left ( 3\sqrt{3}-2\sqrt{2} \right )\left ( 3\sqrt{3}+2\sqrt{2} \right )}+\frac{3\left ( 3\sqrt{3}-2\sqrt{2} \right )}{\left ( 3\sqrt{3}+2\sqrt{2} \right )\left (3\sqrt{3}-2\sqrt{2} \right )}
= \frac{4\left ( 3\sqrt{3}+2\sqrt{2} \right )+3\left ( 3\sqrt{3}-2\sqrt{2} \right )}{\left ( 3\sqrt{3}-2\sqrt{2} \right )\left ( 3\sqrt{3}+2\sqrt{2} \right )}
Using   (a – b) (a + b) = a2 – b2
= \frac{12\sqrt{3}+8\sqrt{2}+9\sqrt{3}-6\sqrt{2}}{\left ( 3\sqrt{3} \right )^{2}-\left ( 2\sqrt{2} \right )^{2}}
= \frac{21\sqrt{3}+2\sqrt{2}}{27-8}
= \frac{21\sqrt{3}+2\sqrt{2}}{19}

Putting the given values,= \frac{21\left ( 1\cdot 732 \right )+2\left ( 1\cdot 414 \right )}{19}
=\frac{39\cdot 2014}{19}
= 2.0632
Hence the answer is 2.0632.

View Full Answer(1)
Posted by

infoexpert27

Simplify :- \frac{7\sqrt{3}}{\sqrt{10}+\sqrt{3}}-\frac{2\sqrt{5}}{\sqrt{6}+\sqrt{5}}-\frac{3\sqrt{2}}{\sqrt{15}+3\sqrt{2}}

Answer. 1
Solution.
\frac{7\sqrt{3}}{\sqrt{10}+\sqrt{3}}-\frac{2\sqrt{5}}{\sqrt{6}+\sqrt{5}}-\frac{3\sqrt{2}}{\sqrt{15}+3\sqrt{2}}
Rationalise the denominators:
\Rightarrow \left ( \frac{7\sqrt{3}}{\sqrt{10}+\sqrt{3}}\times \frac{\sqrt{10}-\sqrt{3}}{\sqrt{10}-\sqrt{3}} \right )-\left ( \frac{2\sqrt{5}}{\sqrt{6}+\sqrt{3}} \times \frac{\sqrt{6}-\sqrt{5}}{\sqrt{6}-\sqrt{5}}\right )-\left ( \frac{3\sqrt{2}}{\sqrt{15}+3\sqrt{2} } \times\frac{\sqrt{15}-3\sqrt{2}}{\sqrt{15}-3\sqrt{2}}\right )
\Rightarrow \frac{7\sqrt{3}\left ( \sqrt{10} -\sqrt{3}\right )}{10-3}-\frac{2\sqrt{5}\left ( \sqrt{6}-\sqrt{5} \right )}{6-5}-\frac{3\sqrt{2}\left ( \sqrt{15}-3\sqrt{2} \right )}{15-8}
\left [ \because a^{2}-b^{2} = \left ( a+b \right )\left ( a-b \right )\right ]
\Rightarrow \frac{7\sqrt{3}\left ( \sqrt{10} -\sqrt{3}\right )}{7}-\frac{2\sqrt{5}\left ( \sqrt{6}-\sqrt{5} \right )}{1}-\frac{3\sqrt{2}\left ( \sqrt{15}-3\sqrt{2} \right )}{3}
\Rightarrow \frac{7\sqrt{30}-21}{7}-\frac{2\sqrt{30}-10}{1}+\frac{3\sqrt{30}-18}{3}
\Rightarrow \frac{21\sqrt{30}-63-42\sqrt{30}+210+21\sqrt{30}-126}{21}
\Rightarrow \frac{21}{21}= 1
Hence the answer is 1.

View Full Answer(1)
Posted by

infoexpert27

NEET 2024 Most scoring concepts

    Just Study 32% of the NEET syllabus and Score up to 100% marks


Express 0.6 + 0\cdot \bar{7}+0\cdot \overline{47} in the form \frac{p}{q} where p and q are integers and q\neq 0.

Answer. \frac{167}{90}
Solution. Let x = 0.6
Multiply by 10 on LHS and RHS
10x = 6
x= \frac{6}{10}

x= \frac{3}{5}
So, the \frac{p}{q} from of 0.6 = \frac{3}{5}
Let y = 0\cdot \bar{7}
Multiply by 10 on LHS and RHS

10y = 7.7777 …….
10y – y = 7\cdot \bar{7}-0\cdot \bar{7}

          = 7.77777 ….. – 0.77777 ……
9y = 7

y= \frac{7}{9}
So the \frac{p}{q} from of 0.7777 = \frac{7}{9}
Let z = 0.47777…
Multiply by 10 on both side
10z = 4.7777 ….
10z – z = 4\cdot \bar{7}-0\cdot4\bar{7}

9z = 4.3

z\approx \frac{4\cdot 3}{9}
z= \frac{43}{90}
So the \frac{p}{q}from of 0.4777 …. = \frac{43}{90}
Therefore, \frac{p}{q} form of 0.6 + 0\cdot \bar{7}+0\cdot4\bar{7} is,

x+y+z= \frac{3}{5}+\frac{7}{9}+\frac{43}{90}
= \frac{\left ( 54+70+43 \right )}{90}
= \frac{167}{90}
Hence the answer is \frac{167}{90}

 

View Full Answer(1)
Posted by

infoexpert27

Show that one and only one out of n, n + 4, n + 8, n + 12 and n + 16 is divisible by 5, where n is any positive integer.

By Euclid’s division –
Any positive integer can be written as:
n = bm + r
Here b = 5
r is remainder when we divide n by 5, therefore:
\leq< 5, r = 0, 1, 2, 3.4
n = 5m + r, therefore n can have values:
n = 5m, 5m + 1, 5m + 2, 5m + 3, 5m + 4.
Here m is natural number
Case 1 : Let n is divisible by 5, it means n can be written as :
n = 5m,
Now, n +4 = 5m + 4;it gives remainder 4, when divided by 5
Now, n + 8 = 5m + 8= 5(m +1) +3; it gives remainder 3, when divided by 5
Now, n + 12 = 5m + 12= 5(m +2) +2; it gives remainder 2, when divided by 5
Now, n + 16 = 5m + 16= 5(m +3) +1; it gives remainder 1, when divided by 5

Case 2: Let n + 4 is divisible by 5, it means n + 4 can be written as :
n + 4 = 5m,
Now, n = 5m – 4 = 5(m – 1) + 1 ; it gives remainder 1, when divided by 5
Now, n + 8 = 5m + 4; it gives remainder 4, when divided by 5
Now, n + 12 = 5m + 8 = 5(m +1) +3; it gives remainder 3, when divided by 5
Now, n + 16 = 5m + 12 = 5(m +2) +2; it gives remainder 2, when divided by 5
Similarly, we can show for other cases.

View Full Answer(1)
Posted by

infoexpert27

Crack CUET with india's "Best Teachers"

  • HD Video Lectures
  • Unlimited Mock Tests
  • Faculty Support
cuet_ads

For any positive integer n, prove that n3 – n is divisible by 6.

Let three consecutive positive integers be, x, x + 1 and x + 2.
Divisibility by 3: Whenever a number is divided by 3, the remainder obtained is either 0 or 1 or 2
.\thereforen = 3p or 3p + 1 or 3p + 2, where p is some integer.
So, we can say that one of the numbers among x, x + 1 and x + 2 is always divisible by 3.
\Rightarrow x (x + 1) (x + 2) is divisible by 3.

Divisibility by 2: Whenever a number is divided 2, the remainder obtained is 0 or 1.
\therefore  n = 2q or 2q + 1, where q is some integer.
So, we can say that one of the numbers among x, x + 1 and x + 2 is always divisible by 2.\Rightarrow
x (x + 1) (x + 2) is divisible by 2.
Since, x (x + 1) (x + 2) is divisible by 2 and 3.
\thereforex (x + 1) (x + 2) is divisible by 6.
Now the given number is:
P = n3 – n
P = n (n2 – 1)
P = n (n – 1) (n + 1)
P = (n – 1) n (n + 1)
Therefore, P is the product of three consecutive numbers.
Now P can be written as:
P = x (x + 1) (x + 2) which is is divisible by 6.
Hence P is divisible by 6.
Hence proved

View Full Answer(1)
Posted by

infoexpert27

Prove that one of any three consecutive positive integers must be divisible by 3.

Let three consecutive integers are n, n + 1, n + 2 when n is a natural number
  i.e., n = 1, 2, 3, 4, ….
for n = 1,         chosen numbers :(1, 2, 3) and 3 is divisible by 3   
for n = 2,         chosen numbers :(2, 3,4) and 3 is divisible by 3    
for n = 3,         chosen numbers :( 3, 4, 5) and 3 is divisible by 3  
for n = 4,         chosen numbers :(4, 5, 6) and 6 is divisible by 3
for n = 5,         (chosen numbers :(5, 6, 7) and 6 is divisible by 3   
Proof:

Case 1 : Let n is divisible by 3, it means n can be written as :
n = 3m,
Now, n +1 = 3m + 1 ; it gives remainder 1, when divided by 3
Now, n +2 = 3m + 2; it gives remainder 2, when divided by 3

Case 2: Let n + 1 is divisible by 3, it means n +1 can be written as :
n +1 = 3m,
Now, n = 3m – 1= 3(m – 1) + 2 ; it gives remainder 2, when divided by 3
Now, n +2 = 3m + 1; it gives remainder 1, when divided by 3

Case 3: Let n + 2 is divisible by 3, it means n +2 can be written as :
n +2 = 3m,
Now, n = 3m – 2= 3(m – 1) + 1 ; it gives remainder 1, when divided by 3
Now, n +1 = 3m – 1=3(m – 1) + 2  ; it gives remainder 2, when divided by 3

View Full Answer(1)
Posted by

infoexpert27

JEE Main high-scoring chapters and topics

Study 40% syllabus and score up to 100% marks in JEE

filter_img